Ặ(Ặ(U)) =uẶ(), Vz, uc R1.

Một phần của tài liệu Tài liệu bồi dưỡng học sinh giỏi Toán phần 2 (Trang 51 - 53)

- Không thể xây ra aƯ; =b với Vi ,7 (Điều này dễ dàng suy ra bằng phương pháp chứng minh phản chứng) ch đó suy ra rằng phải có một số giá trị địy = Ể Mặt khác,

1) Ặ(Ặ(U)) =uẶ(), Vz, uc R1.

2) Ặ(z) Ởể0 khi z Ở +oo.

. Lời giải. Giả sử tồn tại hàm số Ặ(z) thoả mãn yêu cầu bài ra.

` Do Ặ(z) là toàn ánh và 1 Ạ RT nên 3o Ạ RẨ: Ặ(0o) = 1. Trong 1) choz =1, y=

to được

Ặ@) = Ặ.1)/.706)) = Ặ(1) 2 we=1 (do Ặ(1) > 0) = go =1= /(1) =1.

Vậy Ặ(z) có một điểm bất động là z = 1. Ta sẽ chứng minh đó là điểm bất động duy nhất. Thật vậy, giả sử Ặ(z) có hai điểm bất động z, phân biệt, khi đó ta có:

(ự) Ở Ặf(zw) = Ặ(zẶ()) =wf( 5 # = zụ = zụ cũng là điểm bất động của Ặ(z). (1)1=Ặ()= /G/ )=zẶ(_- kh h cũng là điểm bất động của Ặ(z).

Giả sử Ặ(z) còn có điểm bất động + ⁄: 1, khi đó, theo (ii) Ặ(z) còn có điểm bất động =' Trong hai số đó phải có một số > 1. Giả sử z > 1. Theo () Ặ(z) sẽ có vô số điểm bất động zẤ = 7ồ, n Ạ ứ*. Ta có dãy đối số (zẤ) Ở +oo (do z > 1), do đó dãy hàm (Ặ(za)) = (đa) Ở +oo. Điều đó trái với 2): Ở Ặ(z) Ở 0 khi z Ở +ce.

Vậy giả sử của ta là sai, tức là Ặ(z) có duy nhất một điểm bất động là z =

Nhưng trong (¡) cho = z ta được Ặ(zẶ(z)) = zẶ(z), Vxze RẨ => #Ặ(z) cũng là điểm bất động của. Ặ (z). Vậy ta phải có

VzẠR': zẶ(z) =1 ẶỂ) = =.

ể= . , ể - Am

Dễ thây hàm số này thoả mãn các điều kiện của bài ra. Vậy Ặ(z) = : là hàm số cần tìm.

Vắ dụ 9. (Từ IMO 83) Cho các số a, b, cc ứ* , đôi một nguyên tố cùng nhau. Chứng minh rằng phương trình

jC + ca + zab = 2abcỞ abTỞ beỞ ca (1)

không có nghiệm tự nhiên.

Lời giải. Giả sử (1) có nghiệm (z;y; z) Ạ ứở.

Từ (1) ta có (2 + 1)ab:e. Mà (a,e) = (b,e) = 1 = z + 1e, mà z + 1 > 0 nên từ đó

có z+1>ec.

Tương tự, có z+1>a, y+1>b. Từ các đánh giá này ta được

VT7(1) 3 (ỦỞ1)be+(bỞ1)ca+(cỞ1)ab = 3abeỞabỞbeỞca > 2abeỞ abỞbeỞ ca = VP(1) Điều vô lý này chứng tỏ giả sử của ta là sai, bức là phương trình (1) không có nghiệm tự nhiên.

Vắ dụ 10. (Putnam 1998) Chứng minh rằng với mọi số nguyên a, b, c, luôn tìm được số nguyên dương w sao cho số

Ặ(n) = nẾ + an? + bn + c không phải là số chắnh phương.

Lời giải. Ta cần chứng minh mệnh đề sau:

V(a;b;c) ẠZẺ, 3n ứ*: Ặ(n) không phải là số chắnh phương. (?} Nhân xét rằng mọi số chắnh phương đều = 0 (mod 4) hoặc = 1 (mod 4).

Giả sử (p) là mệnh đề sai, tức là:

1(a;b;c) ẠZỢ, VnẠứ*: Ặ(n) là số chắnh phương. (P)

Đặc biệt, từ đó có: Ặ) =a+b+c+l là số chắnh phương(¡) 2) =4a+2b+c+8 là số chắnh phương() Ặ(3) =9a+3b+c+ 27 là số chắnh phương() Ặ(4) = 16+ 4b+c+64 là số chắnh phương(Ư0) Từ () và (iv) ta có Ặ(4) Ở Ặ(2) = 2b (mod 4). Mà 2b là số chẫn, còn theo nhận xét thì: Ặ(4) Ở Ặ(2) chỉ có thể = 0, 1, Ở1 (mod 4) = 2b = 0 (mod 4). Từ (¡) và (1ỉ) ta có Ặ(3) Ở Ặ( = (2b+2) (mod 4).

Tương tự trên ta cũng có 2b + 2 = 0 (mod 4) = 2 = 0 (mod 4) là điều vô lý. Vậy giả sử của ta là sai, tức là (p) là mệnh đề đúng.E3

Một phần của tài liệu Tài liệu bồi dưỡng học sinh giỏi Toán phần 2 (Trang 51 - 53)

Tải bản đầy đủ (PDF)

(80 trang)